Did I use the correct identities in my variation method calculations?

Click For Summary
The discussion revolves around the calculations for a one-dimensional quartic oscillator using a trial function in variational method calculations. The user expresses uncertainty about the correctness of their answer, particularly due to reliance on specific mathematical identities for simplification. They detail their process, including the application of the identities to compute the variational integral W. The final result involves a division of two expressions derived from the identities, leading to a value that the user questions. The thread highlights the importance of verifying the use of mathematical identities in variational calculations.
Emily Smith
Messages
1
Reaction score
0
<Moderator's note: Moved from a technical forum and thus no template.>

Problem:
One dimensional quartic oscillator, V(x) = cx^4 (c is a constant)
Use the trial function e^(-aplha(x^2)/2) to determine the value of the appropriate variational integral W.

I've attached a picture of my work.

I feel that my answer is not correct, because I ended up using identities (listed below) at the end because I was grasping for straws. So my question is: did I mess up by using the identities?

Identity 1 = x^n e^bx^2 = (2n!)/(2^(2n+1)n!)(pi/(b^2n+1))^1/2
Identity 2 = e^-bx^2 = 1/2 (pi/b)^1/2

I used the phi* V phi / phi*phi equation. Started with phi *(V)phi, e^(-aplha(x^2)/2)(Cx^4)e^(-aplha(x^2)/2) dx
I pulled the C out since it was constant and simplified to e^(-aplha(x^2))(x^4)
From there I used the Identity 1 and got 0.0625C (pi/(aplha^5))^(1/2).

For phi*phi I used Identity 2 and got sqrt(pi/alpha)/2

Finally I divided hi* V phi by phi*phi which gave me 0.125C(sqrt(pi/(alpha^5))/ sqrt(pi/alpha)
IMG_20181001_204034.jpg
IMG_20181001_204034.jpg
 

Attachments

  • IMG_20181001_204034.jpg
    IMG_20181001_204034.jpg
    34.2 KB · Views: 309
Last edited by a moderator:
Physics news on Phys.org
Please type in what you've done, but the least, rotate your pictures.
 
Question: A clock's minute hand has length 4 and its hour hand has length 3. What is the distance between the tips at the moment when it is increasing most rapidly?(Putnam Exam Question) Answer: Making assumption that both the hands moves at constant angular velocities, the answer is ## \sqrt{7} .## But don't you think this assumption is somewhat doubtful and wrong?

Similar threads

Replies
4
Views
2K
Replies
3
Views
2K
  • · Replies 34 ·
2
Replies
34
Views
3K
Replies
8
Views
2K
Replies
1
Views
2K
  • · Replies 8 ·
Replies
8
Views
2K
Replies
14
Views
2K
  • · Replies 13 ·
Replies
13
Views
1K
  • · Replies 5 ·
Replies
5
Views
2K